The Stacks project

Theorem 10.34.1 (Hilbert Nullstellensatz). Let $k$ be a field.

  1. For any maximal ideal $\mathfrak m \subset k[x_1, \ldots , x_ n]$ the field extension $\kappa (\mathfrak m)/k$ is finite.

  2. Any radical ideal $I \subset k[x_1, \ldots , x_ n]$ is the intersection of maximal ideals containing it.

The same is true in any finite type $k$-algebra.

Proof. It is enough to prove part (1) of the theorem for the case of a polynomial algebra $k[x_1, \ldots , x_ n]$, because any finitely generated $k$-algebra is a quotient of such a polynomial algebra. We prove this by induction on $n$. The case $n = 0$ is clear. Suppose that $\mathfrak m$ is a maximal ideal in $k[x_1, \ldots , x_ n]$. Let $\mathfrak p \subset k[x_ n]$ be the intersection of $\mathfrak m$ with $k[x_ n]$.

If $\mathfrak p \not= (0)$, then $\mathfrak p$ is maximal and generated by an irreducible monic polynomial $P$ (because of the Euclidean algorithm in $k[x_ n]$). Then $k' = k[x_ n]/\mathfrak p$ is a finite field extension of $k$ and contained in $\kappa (\mathfrak m)$. In this case we get a surjection

\[ k'[x_1, \ldots , x_{n-1}] \to k'[x_1, \ldots , x_ n] = k' \otimes _ k k[x_1, \ldots , x_ n] \longrightarrow \kappa (\mathfrak m) \]

and hence we see that $\kappa (\mathfrak m)$ is a finite extension of $k'$ by induction hypothesis. Thus $\kappa (\mathfrak m)$ is finite over $k$ as well.

If $\mathfrak p = (0)$ we consider the ring extension $k[x_ n] \subset k[x_1, \ldots , x_ n]/\mathfrak m$. This is a finitely generated ring extension, hence of finite presentation by Lemmas 10.31.3 and 10.31.4. Thus the image of $\mathop{\mathrm{Spec}}(k[x_1, \ldots , x_ n]/\mathfrak m)$ in $\mathop{\mathrm{Spec}}(k[x_ n])$ is constructible by Theorem 10.29.10. Since the image contains $(0)$ we conclude that it contains a standard open $D(f)$ for some $f\in k[x_ n]$ nonzero. Since clearly $D(f)$ is infinite we get a contradiction with the assumption that $k[x_1, \ldots , x_ n]/\mathfrak m$ is a field (and hence has a spectrum consisting of one point).

Proof of (2). Let $I \subset R$ be a radical ideal, with $R$ of finite type over $k$. Let $f \in R$, $f \not\in I$. We have to find a maximal ideal $\mathfrak m \subset R$ with $I \subset \mathfrak m$ and $f \not\in \mathfrak m$. The ring $(R/I)_ f$ is nonzero, since $1 = 0$ in this ring would mean $f^ n \in I$ and since $I$ is radical this would mean $f \in I$ contrary to our assumption on $f$. Thus we may choose a maximal ideal $\mathfrak m'$ in $(R/I)_ f$, see Lemma 10.17.2. Let $\mathfrak m \subset R$ be the inverse image of $\mathfrak m'$ in $R$. We see that $I \subset \mathfrak m$ and $f \not\in \mathfrak m$. If we show that $\mathfrak m$ is a maximal ideal of $R$, then we are done. We clearly have

\[ k \subset R/\mathfrak m \subset \kappa (\mathfrak m'). \]

By part (1) the field extension $\kappa (\mathfrak m')/k$ is finite. Hence $R/\mathfrak m$ is a field by Fields, Lemma 9.8.10. Thus $\mathfrak m$ is maximal and the proof is complete. $\square$


Comments (4)

Comment #47 by Rankeya Datta on

I don't know if this counts as a math error, but there seems to be some inconsistency between how the theorem is stated and the way the proof begins. The first few lines of the proof justify why it is enough to prove part (1) of the theorem for polynomial algebras k[x_1,...,x_n] instead of finitely generated k-algebras. But, part (1) is stated in terms of polynomial algebras, and not finitely generated k-algebras. So, I think the statement of part (1) should be modified accordingly.

Comment #53 by on

Well, the beginning of the proof discusses the case where the algebra is an arbitrary finite type k-algebra. See last line statement theorem. I think it is OK (but not great).

Comment #5862 by yogesh on

The proof of part (2) is fine, I just would have been helped with an initial sentence on the strategy of the proof, e.g. "The non-trivial containment we need to show is and we show this by showing the contrapositive of this inclusion statement, namely that for all , if then , which is to say there exists some maximal ideal with and .

Also, the place where one is using is radical is hidden in the claim that is non-zero - if in this ring then and since is radical that would mean contrary to hypothesis.

It may not make sense to include comments on geometric intution within the proof, and obviously you know the following but if I may explain my understanding (hopefully mostly correct) of geometrically what is happening when considering is you are looking at the open set and and then a closed point in has finite degree of by part (1) using (and hence R_f/IR_fk\mathrm{Spec} \ R$.

There are also:

  • 4 comment(s) on Section 10.34: Hilbert Nullstellensatz

Post a comment

Your email address will not be published. Required fields are marked.

In your comment you can use Markdown and LaTeX style mathematics (enclose it like $\pi$). A preview option is available if you wish to see how it works out (just click on the eye in the toolbar).

Unfortunately JavaScript is disabled in your browser, so the comment preview function will not work.

All contributions are licensed under the GNU Free Documentation License.




In order to prevent bots from posting comments, we would like you to prove that you are human. You can do this by filling in the name of the current tag in the following input field. As a reminder, this is tag 00FV. Beware of the difference between the letter 'O' and the digit '0'.